Limit of an integral is from a to a does it always equal to zero?

In summary, the conversation discusses the concept of definite integrals and the process of changing variables in integrals. It is shown that when making a change of variables, the new variable must be defined on every point of the interval of the old variable. If this is not done carefully, the result may be incorrect. Examples are provided to illustrate this concept.
  • #1
vkash
318
1
∫f(x).dx from a(lower limit) to a(upper limit) =0
Is this always true
 
Last edited:
Physics news on Phys.org
  • #2
Yes, for any integrable function f, [itex]\int_a^a f(t)dt= 0[/itex].
 
  • #3
HallsofIvy said:
Yes, for any integrable function f, [itex]\int_a^a f(t)dt= 0[/itex].

I have a question which has really let me think about this. Let me present it.
i want to integrate e-x^2 from 0 to infinity. let me say it I.
How i did it.
(You need a copy pen so that you can write it mathematically.)
step 1 replace x by -x
limits will change from -infinity to zero. rest of the function inside integral will remain same.
step 2 now add I you obtain in step 2 with I in original question.
Now it's function is same but limit is from -infinity to infinity and it is equal to 2I
now replace x by 1/x. limits will from zero to zero.
No matter what the function inside the integral. Now it should zero(i mean I).
so finally i came to know that I=0.
It's really confusing result because if we draw rough curve of e-x^2 then we will find that it is always above the x axis. so how it's integral can be zero since definite integral denote geometrical area under under the curve.
 
  • #4
when you replace x by 1/x, you have a term 1/x2 in the integrand, which is not integrable near 0. Furthermore the limits are not [0,0] but [0,∞] and [-∞,0].
 
  • #5
why not you replying guys. who is wrong here?
I think wolframalpha try to say it is infinity
 
Last edited:
  • #6
vkash said:
why not you replying guys. who is wrong here?
I think wolframalpha try to say it is infinity

The problem is that when you make a change of variables in an integral, the new variable has to be defined on every point of the interval of the old variable. If you have an integral on the interval [itex]t \in (-\infty,\infty)[/itex], you cannot simply make the change [itex]x = 1/t[/itex], because the new variable x is not defined when t = 0, which exists on that interval. Hence, you must split the integral up first, before you can make that change of variables:

[tex]\int_{-\infty}^{\infty} dt~f(t) = \int_{-\infty}^{0} dt~f(t) + \int_{0}^{\infty} dt~f(t).[/tex]

If you change variables now to t=1/x, you still have to be careful, because in the first term (from -infinity to 0), the variable is negative, whereas it is positive in the second term. If you just haphazardly let t = 1/x, then in the first integral you might mistakenly think the upper zero is +infinity, when really it should be -Infinity. You need to account for this, and this is what mathman was telling you. So, after letting t = 1/x, the integrals are really

[tex]-\int_{0}^{-\infty} dx~\frac{f(1/x)}{x^2} - \int_{\infty}^{0} dx~\frac{f(1/x)}{x^2} = \int_{-\infty}^{\infty} \frac{dx}{x^2}~f(1/x).[/tex]

Wolframalpha will confirm that

[tex]\int_{-\infty}^\infty dx~\frac{\exp(-1/x^2)}{x^2} = \sqrt{\pi} = \int_{-\infty}^\infty dt~\exp(-t^2).[/tex]

Any time your change of variables is undefined or multivalued on the interval of the original variable, you need to be careful. Consider the following integral as an example:

[tex]\int_{-a}^a dx~x^2.[/tex]

You know the result is just [itex](2/3)a^3[/itex], but suppose you changed variables to [itex]t = x^2[/itex], giving [itex]dt = 2x dx = 2\sqrt{t}dx[/itex]. If you blindly apply this change of variables you might think you get

[tex]\frac{1}{2}\int_{a^2}^{a^2} dt~\sqrt{t} = 0.[/tex]
The problem, of course, is that the change of variable is different on the [-a,0] and [0,a] intervals. On the negative interval, you need to select the negative root, x = -t1/2, while on the positive interval you need to select the positive root, x = +t1/2. Up above, I 'mistakenly' used the positive root on both intervals. If I am careful, this integral is really

[tex]-\frac{1}{2}\int_{a^2}^0 dt~\sqrt{t} + \frac{1}{2}\int_0^{a^2} dt~\sqrt{t} = \int_0^{a^2} dt~\sqrt{t} = \frac{2}{3}(a^2)^{3/2} = (2/3)a^3,[/tex]
as expected.
 
Last edited:

1. What is the definition of the limit of an integral?

The limit of an integral is a mathematical concept that represents the value that an integral approaches as the size of the intervals used in the integration process approaches zero.

2. Why is the limit of an integral from a to a always equal to zero?

This is because the integral is defined as the area under the curve between two points, and when those two points are the same (a to a), the area is essentially a line with no width, resulting in a value of zero.

3. Is the limit of an integral always equal to zero?

No, the limit of an integral can have various values depending on the function being integrated and the limits of integration. It is only equal to zero when the limits of integration are the same.

4. Can the limit of an integral exist if the function being integrated is not continuous?

No, the limit of an integral can only exist if the function being integrated is continuous. If the function has any discontinuities within the given limits of integration, the limit does not exist.

5. How does the limit of an integral relate to the fundamental theorem of calculus?

The fundamental theorem of calculus states that the derivative of a function can be calculated by evaluating the limit of an integral. In other words, the limit of an integral is the inverse process of taking a derivative, and understanding the relationship between the two can help in solving complex mathematical problems.

Similar threads

Replies
2
Views
282
Replies
16
Views
2K
Replies
31
Views
916
  • Calculus
Replies
12
Views
1K
Replies
8
Views
417
  • Calculus
Replies
8
Views
2K
  • Calculus
Replies
1
Views
1K
Replies
20
Views
2K
Replies
6
Views
2K
Replies
12
Views
1K
Back
Top